Quiz3

  • November 2019
  • PDF

This document was uploaded by user and they confirmed that they have the permission to share it. If you are author or own the copyright of this book, please report to us by using this DMCA report form. Report DMCA


Overview

Download & View Quiz3 as PDF for free.

More details

  • Words: 756
  • Pages: 4
Name: ________________ QUIZ # 3 For the following questions, use the following scenario: A 64 y.o. homeless patient is admitted to the ER because of dyspnea. The patient is disoriented, tachypneic, with low-grade fever. His CBC reveals WBC = 13,000, with 84% PMN, Hgb = 11.2, Chemistry is unremarkable accept for CPK = 50,000, and K=6.2. 1. Which of the following is least appropriate at this stage? A. B. C. D.

E.C.G Chest X-ray ABGs D-dimer

2. ABGs reveal the following: pH – 7.46, pCO2 – 32, pO2 – 55, HCO3 – 22. What is the most likely explanation?

A. Metabolic acidosis, with compensatory respiratory alkalosis B. Simple respiratory alkalosis (secondary to hypoxia). C. Combined respiratory and metabolic alkalosis D. Respiratory acidosis with compensatory metabolic alkalosis 3. Which of the following E.C.G. would you expect the patient to have?

A

B

D

4. What is the most common physical finding of a patient with PE ? A. B. C. D.

Tachypnea Elevated JVP Cyanosis Tachycardia

5. Which of the following conditions cannot explain elevated D-dimer? A. B. C. D.

PE Sepsis MI COPD

? Which of the following ECG criteria does not suggest PE.6 A. Rapid A.fib B. New RBBB C. Inverted T on L3 D. Deep S on L1 E. All of the above suggest PE 7. An 80 y.o. patient is admitted because of nausea and "feeling sick". Her past medical history is IHD, Chronic AFib, and Chronic renal failure. Her E.C.G shows the following – What is your diagnosis ?

A. Atrial fibrillation + intarventricular conduction defect. B. Atrial flutter with alternating block C. Complete AV block with ventricular tachycardia D. Atrial fibrillation with multiple premature beats (APCs) 8. Her Blood Digoxin levels are twice the upper limit. Which of the following intervention is erroneous for her?

A. Reducing her K+ levels with keyexalate. B. FAB antibodies (Antibodies against the Digoxin receptor) C. Dialysis D. Lidocain, if abundant VPBs are present. 9. Which of the following drugs is not appropriate to slow Rapid A.fib (Assuming that the patient is hemodynamically stable)? A. B. C. D.

Verapamil Adenosin Amiodarone Digoxin

10. Which of the following is not appropriate for Digoxin therapy? A. B. C. D.

CHF class III Rapid A.Fib, non-responsive to CCB Rapid A.Fib, with BP 90/60 Acute Pulmonary edema

11. A 50 y.o. is admitted to ER with intermittent chest pain for 5 hours. His past DM, dyslipidemia, smoker. ECG shows ST depression on anterior wall. Troponin – 14. The patient is comfortable, without pain. What is the suitable approach? A. Thrombolysis B. Rescue PCI C. Aspirin, LMWH, and β-blockers

D. Serial ECGs and troponins and stress test 12. Another 60 y.o. patient, without apparent CV risk factors, who regularly exercise,, notes that yesterday while running his 4th mile, a chest pain accrued, which resolved after a short rest. He is asymptomatic. ECG is normal, and troponin = 0. What is the recommended approach? A. Aspirin alone, and Cardiac catheterization B. Aspirin, LMWH, and β-blockers C. Aspirin, LMWH, and β-blockers, until thalium scan D. Aspirin alone, until stress test.

13. A 20 y.o. healthy women comes to the ER with palpitation, and the following ECG. What is the diagnosis? A. Accelerated Idioventricular rhythm B. SVT with abberant conduction C. Ventricular tachycardia D. Atrial flutter 2:1 ventricular response

14. The patient recalls that she had similar episode 2 years ago, and was told that she has "Parkinson" in the heart. The patient looks hemodynamically stable. Which of the following drugs should be used? A. Adenosine B. Amiodarone C. Verapamil D. Sotalol (β-blocker)

15. The patient is hospitalized, and 6 hours later you are called to her. The patient looks pale, dyspneic, Sat = 85%, and bilateral rales are heard on both lungs. Radial pulses are weak, and Monitor shows HR = 210. What is the proper approach? A. Furosamide B. Verapamil C. Digoxin D. Synchronized 100j DC 16. A 60 y.o. patient is admitted to the ER with chest pain and dyspnea. The following arrhythmia is found on his ECG. Which of following intervention would be most appropriate? (Assuming that the patient is hemodynamically stable) A. Lidocain B. Cardioversion C. Verapamil D. Amoidarone

Match between the Chest X-ray and the diagnosis: 17. A 65 y.o with fever and cough for 2 days 18. A 18 y.o with pleuritic chest pain and dyspnea 19. A 70 y.o. hypertensive man with chest pain and newly discovered diastolic murmur. 20. A 60 y.o. rheumatoid arthritis patient with dyspnea and pleuritic chest pain.

A

B

C

D

Related Documents

Quiz3
November 2019 8
Quiz3-result
June 2020 3
Quiz3 Mema
August 2019 4
Quiz3.docx
May 2020 5
Quiz3 72369
October 2019 8
Quizzes > Nm Mech Quiz3
October 2019 6